Basic Calculus help with finding components

Click For Summary
The discussion revolves around solving a calculus problem involving vector displacements and their components. The user presents three displacement vectors and calculates their resultant vector along with the angle between it and the positive z-axis. They express confusion about the method used to find the component of one vector that is perpendicular to another, referencing the Pythagorean theorem. The explanation clarifies that the relationship between the vectors forms a right triangle, justifying the use of the equation for the perpendicular component. The user acknowledges the clarity gained from this understanding.
Zyxer22
Messages
16
Reaction score
0
I always seem to find this place whenever I'm in need of homework help, so I finally decided to register (and hopefully post in the right area).

My given problem is


Here are three displacements, each in meters: d1 = 4.2i + 2.7j - 7.9k, d2 = -1.0i + 2.0j + 3.0k, and d3 = 4.0i + 3.0j + 2.0k. What is r = d1 - d2 + d3 ((a), (b) and (c) for i, j and k components respectively)? (d) What is the angle between r and the positive z axis? (e) What is the component of d1 along the direction of d2? (f) What is the component of d1 that is perpendicular to the direction of d2 and in the plane of d1 and d2?

The solutions I've gotten are

a) 9.2 m
b) 3.7 m
c) -8.9 m
d) 131.91°
e) -6.01 m
f) 7.16 m

All of which I know to be correct. The confusion I'm having is with part f. I used this equation to solve for f:

[(magnitude of d1)^2 - (the component of d1 along the direction of d2)^2]^1/2

I just don't understand why this works. Anyone help?
 
Physics news on Phys.org
This follows the pythagorian theorem. If you draw it to yourself, you'll see the the projection of d1 on d2, and d1 itself, form to vectors, which by connecting their ends form a right triangle. This is generally true for all vectors.
 
I appreciate the answer. And, actually, now that I see it, this should've been really obvious. Thank you ^^
 
Question: A clock's minute hand has length 4 and its hour hand has length 3. What is the distance between the tips at the moment when it is increasing most rapidly?(Putnam Exam Question) Answer: Making assumption that both the hands moves at constant angular velocities, the answer is ## \sqrt{7} .## But don't you think this assumption is somewhat doubtful and wrong?

Similar threads

  • · Replies 6 ·
Replies
6
Views
15K
Replies
1
Views
2K
Replies
20
Views
2K
  • · Replies 3 ·
Replies
3
Views
13K
  • · Replies 3 ·
Replies
3
Views
2K
  • · Replies 8 ·
Replies
8
Views
2K
  • · Replies 3 ·
Replies
3
Views
8K
  • · Replies 11 ·
Replies
11
Views
5K
  • · Replies 8 ·
Replies
8
Views
2K
  • · Replies 6 ·
Replies
6
Views
5K